The Magoosh logo.

GRE Issue Essay: Strategies + 8 Real Student Essays with Scores

The content in this post applies in 2024 to the new, shorter GRE!

When you sit down at the computer on test day, the very first thing you’ll encounter is the GRE AWA Issue essay. For a lot of test-takers, this will feel daunting. But not you! In this article, Magoosh’s experts will guide you through the most important steps in attacking the analyze an issue task. In addition, we’ll take a look at student examples of the GRE “Analyze an Issue” task so that you can understand what gets a high score—and what doesn’t—on the official exam.

gre issue essay - magoosh

Table of Contents

An overview of the gre issue essay.

  • Top 5 AWA Issue Strategies

Student GRE Issue Essay Analysis: Prompts, Essays, and Grading Samples

So, what do you need to do for the GRE AWA Issue essay? Well, your goal is to read the prompt, then agree or disagree with the premise—and explain the extent to which you agree or disagree. Think you can’t prepare in advance? You’d be wrong! There are two main things you can do to get ready for the AWA portion of the GRE.

Review the Topic Pool

First, because the prompts are drawn from GRE’s published pool of Issue Essay topics , a bit of research will give you an idea of what to expect in terms of subject matter and presentation. Don’t try memorize all of them! There’s far too many. But! Do spend time browsing the topics and thinking about how to approach them.

Plan of Attack

Second, come up with a plan to navigate the GRE “analyze an issue” task. Not sure where to start? We can help! Here’s an example of a tried and true process for high-scoring essays that you can use to address any Issue task:

  • Read the directions carefully
  • Brainstorm and outline pros and cons
  • Choose a side
  • Select a concession point
  • Be sure to leave around two minutes for proofreading and editing

magoosh-lesson-video-icon

Top 5 GRE Analyze an Issue Task Strategies

Now that you have the basics down, let’s take a look at some more detailed strategies you can use to maximize your score on the GRE AWA Issue essay.

1. Be Organized

Even an impassioned, cogent response falls apart if it is not bundled into a proper essay format: An introduction, a few body paragraphs, and a conclusion.

2. Focus Your Paragraphs

The Introduction The Intro paragraph has a very limited purpose: The Intro should only introduce the topic and present a clearly defined thesis statement. The thesis will indicate your position on the issue. Your stance should be just ONE of the many points of view about the topic, not more than one. Often it is easiest for the writer—and the reader—if the last sentence in the Intro is the thesis.

The Body Paragraphs The 2-3 body paragraphs make up the bulk of analyzing the issue and should focus on using examples (ideally one per body paragraph) to develop and support your thesis. Make sure you use appropriate transitions and that your sentences link together cohesively so that by the end of each body paragraph you have persuasively—and clearly—shown how your examples supports your thesis.

The Conclusion The conclusion should be very short. In fact, it should only be a few sentences that recap your thesis and supporting points.

3. Keep It Engaging

Repetitive sentence structure makes for repetitive reading. Vary up the way you write—don’t be afraid to use a colon (or a dash), drop in a semi-colon, and vary up the syntax. A constant stream of noun followed by verb followed by adjective implies that you are a hesitant writer. You don’t want the overall impression your essay leaves on the graders to be a resounding meh .

4. Be Specific

Hypotheticals are fine, if you can use them to convincingly back up your point. However, that’s the tough part; “some people,” “mankind,” or “you” are dull and vague. Let’s say you are addressing this prompt: “Knowledge can sometimes be used for destructive ends.” Stating that “Oppenheimer’s knowledge of nuclear fusion allowed him to create the most destructive weapon the world had ever known” is far more impactful than, “scientists can sometimes use technology to hurt us.”

5. Stay On Topic

Perhaps the most important point (lest you wonder why you received a ‘1’ on your essay) is to keep your essay on topic. Imagine you had to respond to the mock prompt on knowledge I used above. If you begin talking about how technology is destructive because smartphones cause us to become insular… you have totally forgotten to answer the question, “Knowledge can sometimes be used for destructive ends.” Address the most compelling examples, yes—but the most compelling examples that relate directly to your topic!  

Now, it’s time to take a look at how sample essays meet (or fail to meet) the above criteria—and how this affected their scores. All of the following essays were written in response to the GRE Issue prompts , so check them out if you haven’t already, and then come back to analyze some examples!

Note: We’ve formatted the essays so that you can see the prompt and instructions first, then try writing your own response (this is great practice!). Once you’ve done that, click on the “essay and analysis” arrows to view examples of graded student essays and see how yours compare.

GRE Issue Essay Prompt 1: University Requirements

Prompt Universities should require students to take courses only within those fields they are interested in studying. Instructions Write a response in which you discuss your views on the policy and explain your reasoning for the position you take. In developing and supporting your position, you should consider the possible consequences of implementing the policy and explain how these consequences shape your position.

Student Essay

Some people believe that universities should put stringent policies in place that require students to take courses only within a chosen field of study, thus harshly limiting the breadth of knowledge that they are able to study. Concentrating on only one field is important in terms of developing expert knowledge and specialization, but it is also crucial that the student hone a well rounded knowledge of the nature of the world so that their field of specialization is accented with courses from outside disciplines as well. It is for this reason that I believe that students should focus their study on a specific field yet also be allowed and encouraged to accent and expand their specialized knowledge by sampling courses from other areas of specialty as well.

Our current globalizing world contains diversity of knowledge, culture and creed that is increasing at a rapid pace and in order to succeed in a world such as this, it is necessary to hone a diverse skill set of knowledge and expertise. Therefore, university policies should encourage students to accent their study of a specific discipline with outside courses that will enhance the breadth of their knowledge about the nature of the world. A student studying medicine, for example, clearly needs to focus the majority of their time on understanding the inner workings of the human body on a scientific level. However, it is also crucial for them to have a more general knowledge of the way in which humans function on an individual or cultural scale (i.e. psychology and anthropology), because effective doctors are not simply capable of diagnosing diseases, but can also interact effectively, with individual and cultural sensitivities, with their patients in order to provide the most well-rounded care. A mathematician who knows only about math and knows nothing about the ancient civilizations whose cultures discovered geography will be ill-suited to make math interesting to his future students or to understand the real world implications of the equations he slaves over daily. A one-dimensional course of study will only serve to foster bias and an uncritical approach to life in such students. Thus, because we live in a world that is multi-faceted, it is important for every specialist to learn a bit about specialities outside of their main discipline in order to augment their understanding of the world at large.

When universities provide a structure of encouragement for their students to augment their specified studies by selecting some courses from outside their discipline, there are some possible consequences, such as the potential for students to change their mind about what they want to focus on. Some may say this is an inefficient use of time and that it will confuse students. However, I would argue that it will foster a wider breadth of knowledge that is ultimately beneficial for any student; a student that started studying biology but then switched to psychology, for example, will always appreciate and pay heed to the importance of our life sciences and will not neglect to consider how the functions of the body may affect someone’s mental health. The existence of knowledge in a wider range of disciplines will only provide the student with more information with which to take charge in a world that is highly complex and rapidly changing all the time, and so allowing them to experiment a little and change their mind once or twice is to their benefit rather than to their detriment.

In conclusion, I disagree that universities should require students to take courses only within their specific, chosen field of study. When students are able to focus their study on one specific topic but then augment it by sampling courses from other disciplines, their knowledge becomes more wide ranging and interdisciplinary, thus providing a better foundation for them to succeed in a rapidly globalizing world. While they may change their minds as to their preferred topic of study one or two times, they will ultimately succeed by having a wide breadth of knowledge that will teach them to approach the world without a subject specific bias. Overall, it is best that universities allow their students to take courses outside of their chosen course of study in order to diversify their pallate of knowledge.

Issue Essay Analysis

This GRE Issue essay starts off with a strong intro that clearly articulates the author’s position. The essay is also very long, and the body paragraphs well developed. In terms of ideas this is a strong—though if slightly limited—essay. It makes a compelling case for interdisciplinary learning. A physician studying anthropology will be more culturally sensitive; a psychologist who studied biology will have a great appreciation for the biological underpinnings of the psyche. The writer justifies this well-roundedness in terms of relevancy: a one-dimensional person will struggle in our complex, globalized world. As well thought out and supported as these points, they are far too similar, and this essay would have benefited from picking another example that argues in favor of allowing students to take courses outside of their majors. Another flaw is the essay doesn’t directly addresses the directions: “should consider the possible consequences of implementing the policy.” Is a world of well-rounded, complex individuals the consequence of allowing students to choose subjects outside of their majors?

Stylistically this essay is not perfect, and I have some minor grumblings.

The ongoing debate about whether a university should require students to take courses only within their fields of study or take extra classes to fulfill graduation requirements is an interesting one. There any many valid arguments to each side and it is not a simple black or white choice when deciding who is right. However, by requiring students to only take courses within their major, it allows for students focus on taking classes that are only applicable to their future careers and allows them to save money in a time where saving money is equally important to a college degree.

In many situations, students will finish high school and go on to college with an idea of what they want to do with their life. For students who are in majors such as engineering or the a science field such as chemistry or biology, it is important to for them to stay on top of all of their course work because of the higher number of courses that they must take in order to fulfill the university requirements for a degree. Many of these students knew before they entered college that this would be the case and gladly accepted that challenge, however by requiring students to take extra general education classes to fulfill their diploma requirements seems counter intuitive to a level of education where students are beginning to focus and narrow in on their future career goals. By forcing say a engineering student to take music theory or British literature just simply to fulfill a general education requirement and having that class conflict with a engineering major course seems to prevent these students from coming to college and fully obtaining their goal as quickly as possible.

The other aspect to consider is the financial aspect. In many of these situations, the students are under pressure to finish their degree as soon as possible because of many state budget cuts to education which limit the number of classes offered with in their major. Not only does this mean extra classes that students must take and thus more money they have to spend because tuition is usually based on a per unit fee, forcing these extra classes upon can have a longer impact if they are forced to stay longer in college than they originally assumed they would. College already charges an extremely large amount to attend and that already does not take into account the other expenses that students have to pay (such as room and board, food, and books), but adding on extra semester, quarters, or even years because a student had to take general education classes instead of strictly major classes is an unfair system to put a student through.

As with any situation though, there are always exceptions to the rule. For one not every student enters college with the same career focus and direction as their peers. Many students will come into college unsure of the direction they want to take and many students who think they know what direction they want to go, end up changing their minds (sometimes multiple times). By requiring students to take classes from a broad range of spectrums, Universities can help students narrow down what career path they may want to follow. Many times students may have a preconceived notion of what a subject may be about and not want to try it, yet by requiring it, they may be able to find themselves in a new class with something they may choose to pursue in the future, something they perhaps never would have considered. There is also something to be said about being able to take higher education classes simply for the benefit of wanting to learn about something that interests you. College allows you to do that and by making it a requirement, it allows students a bigger chance to do that.

Overall though, universities that force students to take upwards of 10-12 general education classes just to fulfill a requirement for their diploma seems unfair. When a student comes into college with a specific end game in site, the universities should not hinder their goals by overloading them with extra requirements and instead focus on helping hem obtain their goals as quickly as possible. The time and financial benefits that could be reaped by not requiring students to take these classes could have a direct impact on the success of all students as well as the future communities they intend to help.

Score: 5.0 This essay covers most of the bases: it offers analysis on both sides of the issue, it throws in a few sentences that address the specific instructions, and it, for the most part, clearly articulates a position. The essay does not wow with thorough analysis, great sentence variety (or indeed any stylistic flourishes). In other words, it gets the job done without making too many missteps.

While I award this essay a ‘5’, there are moments when that score seems shaky. This is not mainly due to the ideas (though the generalizations don’t help: “As with any situation though, there are always exceptions to the rule”); at times the sentences become overloaded and tend to digress.

Word choice could have also been a little more dynamic. “Large”, “bigger”, etc. could be spiced up a little more: “astronomical”, “excessive”, etc.

In addition to making the sentence more readable, and varying up the syntax a little, the essay could have been improved with a little more analysis. I would have like to say more than taking more courses is expensive. Sure, that is a totally valid point, but to spend an entire paragraph on it the overly long first paragraph about students who are not engineers as well.

Additionally, the last body paragraph is confusing: “There is also something to be said about being able to take higher education classes simply for the benefit of wanting to learn about something that interests you. College allows you to do that and by making it a requirement, it allows students a bigger chance to do that.” Is the author implying that colleges shouldn’t require students to take only course in their field (which would go against the main point of the essay)? And by saying that colleges make “it a requirement” that college require students to take courses outside their field?

Had this paragraph been a little clearer and had the writer expanded the scope of the financial issue, this essay—along with a little more dynamic writing and sentence variety—could get at least a definitive ‘5’, if not a ‘5.5’.

Liberal arts colleges and professional schools often debate whether they are required to develop well-rounded individuals. The primary purpose of universities is to establish the ground work for future field experts and specialists, meaning the developing into other fields would detract from the development of specialization. A basic understanding of how to delve into other fields is all that’s necessary.

A college degree in a field suggests that a graduate has the basic understanding of a specialized field, and they may continue to develop into a true expert. At every level of the collegiate process, students have further expansion into their speciality. For instance, science majors start with basic fundamentals that are required for latter learning. They soon go off into their own fields, isolated from the humanities and, often, other science majors. Because students usually have only four years to achieve a set requirement of tested standards in a particular field, universities must push students into their fields quickly. There simply isn’t enough time to truly explore all the possible fields of study at the university level. Exploratory learning shouldn’t be required as it doesn’t serve any purpose when the student won’t continue to explore in those extracurricular fields.

If a student were to only hole themselves away into the fields of physics, they may never truly understand how their physical knowledge relates to society and the social world. Universities tend to have to weigh this “roundedness” against the need to produce future field experts. The outcome is introductory classes that relate to your field, but intertwine with other fields of study, and push students to explore on their own time. These initial exploratory classes would be necessary for any field of study anyway, as creativity and individual pursuit is essential for any expert to further their field’s knowledge.

These exploratory classes are necessary for students to apply their growing expertise, but leaving their fields of study should be done on their own because they can only expand into the elementary levels of other fields within their time restraints at the university level. In this way, students aren’t led by the hand through fields they aren’t interested in, but they would still have the capability to explore their fields if they truly were intrigued. Allowing students to create their own directions, intertwining their interests, creates dynamic individuals who are happier with their degrees and more productive to the world through their specialization.

Universities are meant to develop future experts and specialists in particular fields of study. They should lay the groundwork for students to be able to explore of fields, but not in a way that detracts from their field’s work. At a moment when their time is so precious, students can’t afford to be left behind in their fields as they are forced by curriculum to explore unwanted alternatives.

There are some things about the essay that I like: it brings up interesting ideas relating to the prompt. Do specialists with “roundedness”contribute more to their fields than those specialists who focus only on their fields? The sentence variety makes things flow along nicely, until the middle of the essay, where the author becomes vague. Indeed, at times I’m not sure which side of the prompt the author is arguing.

For example, at the end of the second paragraph he states: “Exploratory learning shouldn’t be required as it doesn’t serve any purpose when the student won’t continue to explore in those extracurricular fields.”

The very next sentence—the first sentence of the third paragraph—says the exact opposite: “If a student were to only hole themselves away into the fields of physics, they may never truly understand how their physical knowledge relates to society and the social world.” Suddenly,the paragraph is arguing against what the previous paragraph stated.

The second to last paragraph is weighed down in abstractions, without a useful specific example to clear things up. Consider the topic sentence: “These exploratory classes are necessary for students to apply their growing expertise, but leaving their fields of study should be done on their own because they can only expand into the elementary levels of other fields within their time restraints at the university level.” There is a lot going on here, and I really had to reread the sentence several times to get what the author was saying. The ETS graders won’t take this much time. And given that the essay has already pulled an about-face in the previous paragraphs, makes this sentence even more obfuscatory.

The conclusion is much clearer than the rest of the essays, and allows me to understand what the essay was trying to say alone.Compare the clarity of this sentence to the one I mentioned in the previous paragraph: “They should lay the ground work for students to be able to explore of fields, but not in a way that detracts from their field’s work.”

So how to grade an essay like this? Strong analytical skills, sophisticated writing, and solid organization….yet, a contradictory—and at times muddled (the clause in the intro, “….meaning the developing into other fields) leads to a confusing essay.

The author states that students should only take classes within their realm of study. Although, students may gain more of a grasp on what they are studying, this requirement fails to take in what students can learn outside of their required classes. To say that students can only take classes within their concentration is occluding them to knowledge that they may learn in other fields of study.

For example, universities typically require students to pick their major, as well as a minor. Some programs may also require students to select a few elective classes as well, so students can establish themselves as more rounded individuals.

Also, taking classes outside of a student’s field of study may help boost the student’s overall GPA. For example, if a student has an in major GPA of 2.5 and an out of major GPA of 3.2, then the overall GPA will increase. However, it could be vice versa as well. If someone isn’t doing that great in their elective classes, it could bring their overall GPA down.

If this policy is implemented, the consequences may be severe. One consequence could be that a student may not be able to graduate on time because they may not have enough credits. Or they may not meet the GPA requirements to graduate because they failed a few classes within their major.

If the university decides that students can only take courses within his or her chosen field of study, then the university may not produce well rounded individuals.

This essay is an example of a 4.0—just barely—that is undeveloped and thus on the short side. It is not an example of a longer, totally one-sided ‘4’ that ignores the directions (notice how the final body paragraph addresses the “consequences” mentioned in the instructions).

What the author has written is an intelligent response to the prompt. She doesn’t simply agree with the prompt, but takes the opposing side, providing support (“To say that students can only take classes within their concentration is occluding them to knowledge that they may learning other fields of study.”). In passing, I should mention that “occlude” is used incorrectly. This is not a major problem, but remember that, if you use GRE words, make sure you know how to use them correctly.

I do not agree with the stated policy to allow students to only take course within their chosen fields of study. Instead I feel that students should should have the opportunity to take course outside of their major for the following reasons.

First, I feel that taken course outside ones major gives students variety, and exposure to experiences or interactions they may not have considered previously. Take for example Lisa, an engineering student who spends countless hours studying. Realizing that she needed a change of place an outlet of some sorts decides to take a modern dance course just for fun. What ultimatly was that Lisa learned to relax which interned helped her study more effectively and perform better in her engineering course.

Then take Monique, a political science major who doesn’t know how to swim. decided to take a swimming course and not only learned to over come her fear, but gained confidence in other other aspects of o her live.

Thirdly, lets consider Jason, a physics major who only took courses in his major. He became such an expert in his field us study, but became increasing socially award because of his inability to converse or relate to his peers.

In the even both Lisa and Monique were not able to take course outside of their major, I fear that they would have succumb to the pressure that sometimes too often over takes students adjusting to university lift. By deviating from their mandatory set of course they found a renew focus and inner strength that they may have never know before. Jason however, didn’t fair as well due to his strict focus in University

University is about diversity and gaining new experience for growth and development. Not being allowed to explore this diversity limits the over experience and potential stunts the education growth and perspective of students

Grammatical errors and spelling mistakes mar the effectiveness of this essay. Specifically, commas are misused (or not used at all), incorrect words are used (“interned”, “award” vs. “awkward”). I think many of these mistakes can be remedied if the student spends some time editing.The point in editing isn’t to catch the nitpicky errors but the glaring ones (of which this essay has many).

Next, the essay has very predictable development: take one-side of the prompt, and then come up with three hypothetical examples to support the point. There is zero analysis. This essay could have been improved and gotten within striking range of a ‘4’, or at least a ‘3.5’, had it simply addressed the instructions: “consider the possible consequences of implementing….” Of course, addressing the grammatical and spelling errors would have helped the essay.

Prompt 2: Lasting Legacy

Prompt Those who see their ideas through, regardless of doubts or criticism others may express, are the ones who tend to leave a lasting legacy. Instructions Write a response in which you discuss the extent to which you agree or disagree with the statement and explain your reasoning for the position you take. In developing and supporting your position, you should consider ways in which the statement might or might not hold true and explain how these considerations shape your position.

A famous author once remarked that “Winners never quit and quitters never win”. People who see their ideas through, however unpragmatic it may be considered by others are the ones who have truly made a difference.

History is replete with examples of people who were perceived as crazy, illogical and even insane by laymen, yet when their ideas were sedulously worked upon, by the creator , day after day, combined with long hours of toil, the result was nothing, short of marvelous.

Lets’s take the example of the Indian freedom struggle fought by Gandhiji on the basis of Satyagraha. It was very difficult for the Britishers to assume that India would be freed one day under the leadership of a loin cloth covered ordinary looking man without the use of weapons or bloodshed. The reason that Indian freedom could be achieved was the unflagging determination of Gandhiji and the uncommon methodology used of winning freedom by peace and not bloodshed.

Looking not far, I can recall the example of Galileo who was reviled and persecuted by the Church authorities for challenging the existing norms that pervaded the society that time. Galileo’s fierce determination , not to give up on his ideas even during harsh criticism paved the way for modern space research.

Another convincing example is of the Wright Brothers. Who would have ever imagined that it is indeed possible to fly like a bird and traverse different parts of the globe. I am sure that the Wright brothers were reviled when they first came up with this idea of developing an aeroplane. But, again today their invention has become a legacy.

Though there are several examples of people winning through odds because of their determination and unflagging spirit and creating noteworthy inventions, there could be times when this may be the cause of much trouble. Consider the doggedness of Hitler.though he was criticised for his heinous atrocitities on the Jews, he still did not stop the atrocities. These are few examples when people with strong determination can create an ill legacy instead of a legacy.

The writing in this essay has a lot of punch and makes reading it easy. However, there is little to no analysis. Like many essays on this prompt, the essay takes an extreme position, and beyond a vague, jumbled mention of Hitler, does not address the instructions: “…you should consider ways in which the statement might or might not hold true.”

As an SAT essay goes—basically you can take a relatively strong position—this is a good essay. Even then, some of the examples lack persuasiveness: “I am sure the Wright brothers were reviled.” Maybe they weren’t (they actually were, somewhat), but to say “you think” vs. “many notable scientists mocked the Wright Brothers notion of human flight” makes the essay far more tentative than it should be.

Also, the examples are very sparse, especially Galileo. Some more development would have perhaps bumped this essay to a ‘4.5’. But without any analysis, and by failing to take into account the other side, this essay gets only a ‘4.’

Although, doubts and criticism expressed regarding a particular by others seem valid at the particular time of inception of time, if the person follows through his idea or well cherished dream, then he may become success in his endeavor and leave a lasting legacy. So, people who see their ideas through, regardless of doubts or criticism others may express, are the ones who tend to leave a lasting legacy.

New ideas takes time to be accepted by general public, and during the time from the inception till the acceptance, the person who invented or discovered that idea, may be criticized or oppressed. Galileo was put into house arrest for his entire life for his heliocentric model of the solar system, because it came in direct conflict with the church’s geocentric model which regarded Galileo’s theory as heresy. Later, Galileo’s model was readily accepted. So, it’s really important that the people should see their ideas through criticism and doubts of others and shouldn’t be daunted, since other people are not connected to the idea or dream or feel the strength of idea in the same way as the person who invented that idea.

If a person doesn’t

This essay struggles from a lack of clarity. The first two sentences are overloaded with words, and so it is difficult for a reader to figure out what the writer is trying to say. Since the essay graders do not have time to figure out what you are trying to say, you will be penalized. Luckily, the thesis is clear—though it is an almost exact rewording of the prompt.

The Galileo example—while expressed in language that is clearer than that found in the intro—isn’t that developed. We learn that he was arrested and confined for heresy. The essay automatically assumes that this is the same as criticism. I would say the church’s actions against Galileo are a little stronger than mere criticism.

What saves this essay from a sub-3.0 is the final sentence, which discriminates between the person with the idea and those who only have an inkling of that idea. However, this idea is not explored in more depth (and doesn’t really connect to the Galileo example). Indeed the essay ends there.

Prompt 3: Risky Action

Prompt People should undertake risky action only after they have carefully considered its consequences. Instructions Write a response in which you discuss the extent to which you agree or disagree with the recommendation and explain your reasoning for the position you take. In developing and supporting your position, describe specific circumstances in which adopting the recommendation would or would not be advantageous and explain how these examples shape your position.

People should undertake risky action only after they have carefully considered its consequences.

People should not let their fears prevent them from taking important risks in life. Taking risks is what allows us humans to achieve success, joy and ultimate fulfillment. However, prior to taking any risky action, it is essential that people should carefully consider the consequences.

For example, there are some risky actions that are life-threatening such as skydiving. Of course, before you can begin to skydive, you must learn the basics of this sport. Additionally, by also studying what can go wrong during a skydive, and learning how to react to that scenario, that person will have the knowledge and ability to stay calm and hopefully make better decisions that will allow them to get out of a bad situation rather than falling into a panic.

This also pertains to decisions about money and business. Everyday people are making decisions that are ‘make or break’. For those who really understand the consequences of their actions, they are able to make a wiser decision that may have less of an impact on them if the business or investment deal goes awry. However, but not educating oneself, the consequences of one’s action are likely to be more severe.

Sometimes, knowing the consequences of an action causes fear that will stops us from taking any risky actions. As a result we miss out on potential successes and most of all “joy”. Therefore, by understanding the consequences, one can eliminate feat, learn how to react in a smarter fashion and lead a much more enriching life than if they had never taken those risks at all.

This is a decent skeleton of an essay. But that’s the problem—it is only a skeleton and the ideas need a lot more fleshing out if this essay is to get at least a ‘5’. For instance, in the skydiving example, the writer barely scratches the surface. What are some things that a skydiver could possibly learn to help them make this risky endeavor less risky? How much less risky would they make sky diving? Is there a point where something is so risky that even if we take measures to prevent disaster from happening that something bad could still happen (skydiving in bad weather, or bungee jumping in a country that offers low prices—and also low quality equipment). In calculating risk, shouldn’t we also weigh the payoff. For the skydiving example, is the thrill worth the danger, even if one has taken the necessary precautions and learned proper technique.

A Final Word

Now that you’ve reviewed student samples from across the spectrum of GRE Issue task grades, you’ll have a better sense of what you need to do to get those high scores! More than anything, practice will help you get the score you want on test day. So take a look at the Issue pool and a few more essay examples, pull up a blank document, and get practicing!

Also, you can now sign up for Magoosh Premium Plan today to access our AI Expert Tutor that will grade your essay and give pointers on how to improve your score.

Chris Lele

Chris graduated from UCLA with a BA in Psychology and has 20 years of experience in the test prep industry. He’s been quoted as a subject expert in many publications, including US News , GMAC , and Business Because .

View all posts

More from Magoosh

Top GRE Scores - image by Magoosh

Leave a Comment

Please leave any questions or suggestions in the comments, we try our best to respond within a few days! Your email address will not be published.

Leave a Reply Cancel reply

Your email address will not be published. Required fields are marked *

49 responses to “GRE Issue Essay: Strategies + 8 Real Student Essays with Scores”

Supatat Hovanotayan Avatar

Hello Magoosh team

First of all, thank you for your amazing tips about the issue essay.

But I still have a question about this task “if I write only two paragraphs, and mainly focus on only one side” Can I still get at least 4 points by doing this

Thank you very much for your kindness and time

Magoosh Expert

Hi Supatat,

By “two paragraphs”, do you mean two body paragraphs? You should aim for an introduction, 2 body paragraphs, and a conclusion. In other words, you should have 4 paragraphs. 🙂 As long as you use strong examples and make your point very clear, you should be able to get 4 points on the exam even without a third body paragraph.

Akshata Lolayekar Avatar

When giving examples whilst supporting our point, can we mention an borrowed idea or opinion and elaborate on it in our own words? Let’s say I mention an idea from Yuval Noah Harari and credit him? Will this be considered plagiarism in any way

Hi Akshata,

You can definitely mention an opinion as long as you state the original source. For example, you can say: “According to Yuval Noah Harari, […]” and that would be acceptable. 🙂

Bayenah Al-shami Avatar

Hello Firstly, thank you for this wonderful article. I have a question which is: How can I say a concession point without making any contradictions to previous paragraphs? I hope that my question is clear. Thanks

Hi there! Thank you, we’re glad you found it helpful. 🙂 I’d recommend reading over the example essays in this blog post to see how they handle the concession point. In addition, be sure to check out our blog article 12 Tips to Ace GRE Writing as well.

Joe Bouzide Avatar

I have a question regarding where to include the concession point in my essay. Does it receive its own paragraph within the body of the essay, or does each supporting idea have a concession point paired with it? And do you include the concession point in the intro and conclusion as well?

Thanks, Joe

Hi Joe! You can add a third body paragraph that discusses your concession if you have time, but you can also just make a quick concession point, say at the end of your second body paragraph. Just remember that the goal is to use the concession to prove your point. The most common mistake is to spend too much time on the concession, so it can be safer to do less than more. I would not recommend bringing up a concession in the introduction or conclusion. It’s possible, but it’s just too risky. Use your concession to say, “While it may seem that people are distracted by their cell phones, they are actually socializing while looking at their screens. Therefore, technology brings people together.” Something like that is a strong, quick concession, whereas if you spend a paragraph going on and on about how people never talk anymore, you run the risk of arguing for the other side! Hope that helps 🙂

Mursal Rabb Avatar

Hi, It is OK to write issue essay from first person perspective?

There is no specific prohibition of the first person and some people do well on the essay and use the first person. But I tend to recommend avoiding first person language, especially “I think” and “in my opinion.” Both of these phrases tend to be redundant because you usually can take these phrases out of the sentence and your sentence will still maintain its meaning and grammar. You can completely avoid the first person and your writing will likely end up with a more sophisticated tone.

If you do use the first person, I’d recommend that you use it once in the introduction paragraph for your thesis, and that is it.

I recommend taking a look at some of the sample essays written on some topics. These are released by ETS, the testmakers, and will give you an excellent idea of what a great, good, and poor essay will look like. You’ll notice that the essays rated 5 and 6 do not have first person language but the other, lower scored essays do.

http://www.ets.org/gre/revised_general/prepare/analytical_writing/issue/sample_responses

Avinash Avatar

I guess I am a lot of thoughts to put on, but facing trouble to make my writing more persuasive. Can you please suggest how i can make my writing more persuasive as to better reflect my thoughts.

In the AWA issue Essay, being persuasive is all about using evidence. Anytime you make a claim, think of the reasons people might doubt that claim. Address all of those most obvious doubts. Also think about any questions people might ask you to get a better idea about what you’re saying in your essay, and why you’re saying it. Always put forth a very complete set of supporting details and argumentative evidence.If you think you won’t have the time or space to complete your argument within the time and pace limits of AWA, then choose a different argument, or find a way to simplify your argument.

Meredith Avatar

One set of directions states to “discuss the extent to which you agree or disagree with the statement…” I’m confused by “extent.” Does this mean that ETS simply wants us to take a side either in agreement or disagreement and explain why? Or by “extent” do they mean that it is okay to strongly disagree, or to somewhat agree, etc.

Hi Meredith,

The second option is more accurate–another way to think about “extent” is “degree.” So not only do your agree or disagree, but what are the limitations of that opinion? I hope that helps! 🙂

Lid Avatar

Can you write in first person on either GRE essays?

There is no specific prohibition of first person and some people do well on the essay and use the first person. But I tend to recommend avoiding first person, especially “I think” and “in my opinion.” Both of these phrases tend to be redundant. You usually can take these phrases out of the sentence and your sentence will still maintain its meaning and grammar. So, you can completely avoid first person and writing in a more sophisticated tone.

If you do use first person, I’d recommend that you use it once in the introduction paragraph for your thesis. And that is it.

I recommend taking a look at some of the sample essays written on some topics . These are released by ETS, the testmakers, and will give you an excellent idea of what a great, good, and poor essay will look like. You’ll notice that a 5 and 6 do not have first person but the other lower scored essays do.

I hope that helps! 🙂

Alyssa Avatar

Hi Chris! I have a questions about the intro paragraph/thesis statement. Do you have to include the points you plan on discussing in your body paragraphs in your intro/thesis?

It’s not necessary to state your points verbatim in your intro — in fact, it will probably save you time not to do so 🙂

Davut Avatar

My exam is on 13th February and I have about 1 month from now on. I tried to focus on verbal and math section more until now and did not spend enough time on AW section of the GRE. Would you recommend writing one essay per day to gain acceleration on practicing ?

Any suggestion would be appreciated. Thanks.

I am so sorry this didn’t get answered quickly, but hopefully our advice can help! I’d suggest that you first take a look at these ETS topic pools:

List of AWA Issue Prompts List of AWA Argument Prompts

Familiarize yourself with these topics, and then write several practice essays of your own using these ETS topics as a way to familiarize yourself with the questions and expectations. If you are careful to answer the actual question posed by the AWA tasks and you prepare yourself by knowing what will be expected of you on that day, you won’t have any trouble getting a good score. 🙂

Laura Avatar

Oppenheimer used nuclear fission, not fusion. 🙂 The GRE grader do not care if your facts are correct, though.

Alex Avatar

Dear Chris,

Firstly, thanks for keeping up with the blog. It’s been a great help.

Secondly, I was wondering if there is any way to insert special characters on the Gre essay software during the exam – such as those required in ‘vis-a-vis’ or ‘blase’ or ‘cliche’. If not, should these phrases/words be avoided? I’m from India and keyboards here don’t have these characters on them by default.

Chris Lele

That is a good question. I have no idea of the keyboards here allow you to do so. Regardless, I don’t think ETS will hold that against you. Of course, there is a computer grader, but maybe it has been programmed not to dock. Still, I can’t image ETS being so picayune as to do you for not having the proper diacritic.

Hope that helps!

Cornelia Avatar

One thing that concerns me when writing my essays in the issue part is that a lot of the examples that come to my mind are not that well-known in the Anglosphere. I’m German, and I often think of something German scientists or politicians did or said, events that happened in Germany or things taught in German high school. The example essays that I compare my essays to usually score high by drawing on a wide range of examples that are well-known in the US. Stating my examples, that the examiner has possibly never heart of, either requires a longer explanation, for which I don’t have time, or googling on part of the examiner.

What would you suggest? In theory, the GRE should not be culturally biased. But I am afraid if I simply drop unknown German examples, the examiners might be confused.

Thank you for your advice,

PS: To know what I mean, I thought of some examples for you. Let’s say the issue is about privacy and I refer to the surge in users of the Posteo.de email client, a Berlin-based start-up whose unique selling point is that they protect their clients’ privacy as much as possible. Or in an essay about rebellion I could refer to the way the German authorities dealt with house occupiers in Dresden in contrast to those in Berlin after the fall of the Berlin wall – the occupiers in Dresden were given proper rent contracts while those in Berlin were forcefully evicted, causing violent clashes with the police. Or when writing about technology, I might want to cite the website dawanda.de where people sell self-crafted goods. I know that there exists a similar format in the US – etsy – but I am not that familiar with it and would not feel comfortable writing about it and would prefer the German example. This issue comes up for me with almost every essay I write at least once!

Holing Avatar

I am on the same boat and would love to see this question answered!

Hi Holing and Cornelia!

I know this is a late reply, but hopefully it can help others in your positions. 🙂

It is perfectly fine to use non-US examples for the GRE essays, but you want to make sure you give relevant context and information on the events so that the reader doesn’t have to guess whether or not your example really applies to the point you are trying to make. If you can do that, then any examples from your own country should be fine. 🙂

Karishma Avatar

Hi, I have read in most sites that practicing essays is the best way to go for AW. But writing a full length AW issue essay or argument essay takes 30 mins each for a time limited atmosphere. So my question is while practicing from the ets pool of topics, do we need to write full length essays for every topic or just structuring and brainstorming on the topic and writing mock essays 3-4 times will be enough?

Margarette Jung

Hi, Karishma

30 minutes for each essay can definitely be tough to fit into your schedule! Doing quick structuring/brainstorming is a good alternative when you don’t have a lot of time. However, especially as you near your exam date, make sure to sit down and do a few full-length essays (not all in a row, but maybe one every few days) just so you can feel comfortable with the experience. I hope that helps! 🙂

Best, Margarette

Thanks Margarette!!

Hashim Avatar

Hello people of Magoosh,

I have a question about writing a thesis for an issue task. I noticed that in the video lesson, the thesis contained a statement indicating choosing a side. However, there’s no mention of the main points covered in the body paragraphs. Is that a good practice? Don’t you think that a reader ought to know what to expect in the body paragraphs just from reading the thesis statement?

Referred thesis: “a college curriculum should be designed around the career a student will pursue upon graduation”

Kevin Rocci

Excellent question! In a typical, untimed essay you definitely would want to let the reader know what is coming. The intro and thesis should give the reader some idea of where the discussion is headed and what will be discussed. This is a common practice in American essay writing.

But with the GRE, our strategies are a little different. Since we have such a limited amount of time to write an essay, we recommend spending as little time as possible writing the introduction and conclusion. The bulk of your time should be spent crafting the body paragraphs. As such, we only recommend stating your opinion or stance on the topic and not worry about prefacing your examples and reasons.

This isn’t to say that you can’t do this. If you are a quick writer and have the time, then you can definitely indicate what the main points of your body paragraph will be. 🙂

Happy Studying!

Lara Avatar

I just started practicing the AWA and am following the 90-day study plan for beginners. I’m trying my best to follow the outlined time structure you suggested in the videos, but in my first two essays I’ve always run out of time and always seem to produce mediocre work. Would you recommend that I practice writing without a time limit for now? Or should I just keep working with the time limit and would I gradually improve with more practice?

Hi Lara, Happy to help!

First, I recommend to keep practicing. Writing the essays on the GRE is a particular type of skill that needs lots of practice. So keep your head down and keep at it.

Second, if you feel like you need extra practice, try writing an essay more often. Instead an essay a week, write two. This will give you more opportunities for improvement.

Third, I recommend that you keep timing yourself. It doesn’t help to be good at writing an essay in an hour. We need to be good at writing an essay in half an hour.

One thing that I have done with my students in the past is have them write only an introduction or only an introduction and body paragraph in a set amount of time. So give yourself a time limit of 8 minutes and see if you can complete an introduction and body paragraph. This allows you to practice writing under time constraints and you can take baby steps towards completing an essay in 30 minutes.

I hope that this helps! Best of luck in your studies! 🙂

Marcel Avatar

I just started reading the book you recommended: On Writing Well, by William Zinsser. Although I would love read all of it, I don’t have much time to spare. Could you suggest what chapters would most benefit us for the GRE AWA ?

Good question! I think the grammar-related passages are important. As are the chapters that relate to crafting sentences and creating paragraphs.

Asma Maladwala Avatar

Hi Chris, Do you know if there are any sites where I can find high scoring sample essays? I’ve been practicing but feel as though I’m in a void as I have no point of comparison. Getting feedback from family and friends is helpful, but I’d just feel so much better if I could compare my essays to actual GRE essays. I could only find one sample set on the ets website…

It seems that only gre.org offers example essays. Just google “example GRE essays” and it should be the second hit.

Besides that there aren’t too many others I can think of that are online. Writing higher scoring essays, ‘5.5-6’ for blog posts is something I plan to do soon though :).

Veronica Avatar

Hello! I would enormously appreciate if you can clarify me this. Which link are you referring to in the following sentence?:

” For practical advice on practicing: the link below provides access to hundreds of essay prompts by ETS”.

I cannot find it anywhere and it would be of invaluable help for me to have these essay prompts in order to practice.

Thank you very much!

No problem :).

Here is the link: http://www.ets.org/gre/revised_general/prepare/analytical_writing/issue/pool

Verónica Avatar

Thanks for your quick response!

J Avatar

So I just found out ETS has started employing their e-rater technology. Thoughts?

Thanks for reporting that! Well, I hope it is better than the GMATs, which apparently counts number of words, a couple of transition sentences, etc. I guess time will tell.

emma Avatar

whats e-rater technology, mentioned by J, Chris??

Muhammad Usama Khan Avatar

Sometime it seems that we cannot write enough in the issue task.

If we practice one essay per day, who will rectify this and will tell us how to improve our score in analytic. So that we can BUT ALL feel confident to write essay with positive tone.

Yes, that is true, and indeed I need to write another post on generating ideas.

As for somebody to give you feedback, find a trusted family member or friend. Of course, that person would not want to read everyone of your essays, but as long as you get feedback every once in awhile that will help :).

Bhavin Parikh

This sentence is dead-on, “If you think you did poorly on the essays, that knowledge could very well affect your performance on the rest of the test.”

I recently talked with a student who was consistently scoring in the 80th percentile on math and verbal in practice. But he wasn’t prepared for the writing section on test day and it affected his concentration throughout the rest of the exam. He scored in the 60th percentile. Doing well on writing can definitely set a positive tone for the rest of the exam.

Yes, I am happy to hear that student’s experience echo my thoughts. Really, “Doing well on writing can definitely set a positive tone for the rest of the exam” is perhaps the greatest GRE tip that nobody has ever heard of.

typeR Avatar

Did u mean non-native below?? “Two of the preeminent prose stylists of the English-language novel were both native-English speakers.”

Ha! Yes, I definitely did. Thanks for catching that :).

Testimonials

Free Resources

PrepScholar GRE Prep

Gre prep online guides and tips, 4 top-scoring gre sample essays, analyzed (issue + argument).

gre issue essay pool ets

The best way to figure out how to get a high Analytical Writing score is to look at a GRE essay sample, but doing so without any guidance can be overwhelming. How do you show insight? Do typos affect your score? What’s a good way to keep your essay organized?

We’ll answer all these questions for you (and more!) in this article by analyzing four real GRE essay examples and highlighting the key features you’ll want to include in your own essays.

How to Use This Guide

Before we get to the GRE sample essays and their analyses, I’ll highlight two best ways to use this guide to improve your essay and get a great scoring essay yourself.

First, use the perfect-scoring sample GRE essays in this guide as models of possible ways to accomplish the essay tasks . By this, I don’t mean you should plagiarize entire sentences, paragraphs, or essays – that’s both wrong and against GRE code of conduct (it will disqualify your entire test if discovered). Plus, there are so many prompts (152 Issue prompts and 176 Argument ones) that it’s unlikely you’d be able to use any of these exact essays anyway.

What you can and should do is incorporate the features highlighted in the analyses below in your own essays. For instance, if you’ve been struggling with how to logically connect ideas within paragraphs in your own essays, take a look of some of the examples of logical connection I point out in this article and see how they fit within the context of the full essay. You can then practice replicating successful connections between ideas in your own practice essays.

The other main way to use this guide is in conjunction with the essay grading rubrics to help ferret out your writing weaknesses and work on them. Start with the rubrics for the Issue and Argument tasks and identify which criteria are most difficult for you to meet. Even if you can’t articulate precisely what your weakest spot is (e.g. failing to logically connect your ideas within paragraphs), you can at least narrow down the general rubric area you most struggle with (e.g. organization in general).

Once you’ve identified the general area you have the most trouble with, read the GRE essay examples and our analyses in this article to find concrete instances (rather than the abstract descriptions) of the rubric criteria. For more information about the different rubrics for the different essay tasks, read our articles on how to write perfect-scoring GRE Issue and Argument essays .

Quick side note: we've created the world's leading online GRE prep program that adapts to you and your strengths and weaknesses. Not sure what to study? Confused by how to improve your score? We give you minute by minute guide.

You don't NEED a prep program to get a great GRE score. But we believe PrepScholar is the best GRE prep program available right now , especially if you find it hard to organize your study schedule and don't know what to study .

Click here to learn how you can improve your GRE score by 7 points, guaranteed .

body_noteyourweaknesses

Because this article is on the longer side, we’ve created a table of contents to enable you to jump to a specific essay example or task type:

Table of Contents: GRE Essay Examples 

Issue essay 1: technology and human ingenuity, issue essay 2: cooperation vs. competition, argument essay 1: mason city riverside recreation, argument essay 2: super screen movie advertising.

The first of the GRE sample essays we’ll be looking at is written in response to the following “Analyze an Issue” prompt:

As people rely more and more on technology to solve problems, the ability of humans to think for themselves will surely deteriorate.

Discuss the extent to which you agree or disagree with the statement and explain your reasoning for the position you take. In developing and supporting your position, you should consider ways in which the statement might or might not hold true and explain how these considerations shape your position.

The essay written on this Issue prompt takes the position that rather than hindering our abilities to think for themselves, technology will spur humanity on to achieve ever-greater things. The full text of this GRE essay sample can be found on the ETS website .

In this analysis, I’ll go over the different ways in which this essay meets the GRE essay rubric criteria for a perfect scoring Issue essay . The first of these rubric criteria I’ll be discussing is the way the author takes a clear and insightful stance on the issue in the essay.

The author’s position that instead of fearing new technology, we should embrace its possibilities is methodically articulated over the course of the entire essay, culminating in the essay’s conclusion with a full thesis statement (“There is no need to retreat to a Luddite attitude to new things, but rather embrace a hopeful posture to the possibilities that technology provides for new avenues of human imagination.”). Below is an outline of how the author expresses her thesis throughout the essay:

  • Paragraph 1 : The author acknowledges “technology has revolutionized the world.”
  • Paragraph 2 : The author explains the reasoning behind the statement in the prompt (“The assumption is that an increased reliance on technology negates the need for people to think creatively to solve previous quandaries”).
  • Paragraph 3 : The author counters the reasoning she discussed in paragraph 2, writing that “reliance on technology does not necessarily preclude the creativity that marks the human species.”
  • Paragraph 4 : The author advances her counterclaim one step further, stating that “technology frees the human imagination.”
  • Paragraph 5 : The author further develops the idea from Paragraph 4, stating “By increasing our reliance on technology, impossible goals can now be achieved.”
  • Paragraph 6 : This final paragraph concludes the essay with a fully articulated thesis that also sums up what went before: “There is no need to retreat to a Luddite attitude to new things, but rather embrace a hopeful posture to the possibilities that technology provides for new avenues of human imagination.”

The author’s straightforward explanations of her thinking and logic enhance the clarity of her position, while the nuanced content of the position itself demonstrates insight into the issue.

body_makingtheimpossiblepossible

The next area a perfect-scoring Issue essay must demonstrate mastery of is the development of its position through compelling and persuasive examples and reasoning . The author of this essay accomplishes this task by providing examples to support each idea she discusses and, furthermore, explaining not only the content of the examples but also why the examples support her position.

Here’s an example from paragraph 5:

By increasing our reliance on technology, impossible goals can now be achieved. Consider how the late 20th century witnessed the complete elimination of smallpox. This disease had ravaged the human race since prehistorical days, and yet with the technology of vaccines, free thinking humans dared to imagine a world free of smallpox. Using technology, battle plans were drawn out, and smallpox was systematically targeted and eradicated.

In this example, the author begins by laying out the main idea to be discussed (impossible things can be achieved by relying more on technology). She then supports this idea with the example of the impossible problem of smallpox and the steps taken that led to its eradication.

The great thing about the way the author explains her reasoning and examples is the concision and precision with which she gets her information across. Rather than going off into a discussion about the damage caused by smallpox, or staying too vague by mentioning how “diseases” had been solved by the use of vaccines, the author chooses a specific example (smallpox) and mentions only the details relevant to proving her point . This kind of precise writing takes practice, but being able to effectively sum up an example and why it supports your position in just a couple of sentences is essential if you want to get a high score on the GRE Issue essay.

body_supportsyourposition

Focus, organization, and logical connections are the third criterion that a perfect-scoring essay needs to fulfill. In the case of this GRE essay sample, the author achieves this organization and focus by linking ideas both within paragraphs (as seen in the previous example) as well as between paragraphs . Let’s look at the way the author transitions between the end of paragraph four and the beginning of paragraph five:

The unlikely marriage of economics and medicine has healed tense, hyperinflation environments from South America to Eastern Europe.

This last example provides the most hope in how technology actually provides hope to the future of humanity. By increasing our reliance on technology, impossible goals can now be achieved.

The author connects the two paragraphs by continuing paragraph four’s discussion of ways human imagination has been pushed by technology (technology combining economics and medicine has solved a problem) with paragraph five’s exploration of how this example has led to achieving things previously considered impossible. The smoothness of the transition between the two paragraphs is effected both by presenting the content of the next paragraph as a logical progression from what was just discussed as well as by using language (“this last example”) that connects the two on a more superficial level.

By keeping paragraphs tightly linked on both the surface level of sentence structures as well as on the deeper level of content being discussed , the author of this essay also keeps her writing focused and cohesive.

body_cohesiveessay

Want to improve your GRE score by 7 points?  We have the industry's leading GRE prep program. Built by world-class instructors with 99th percentile GRE scores , the program learns your strengths and weaknesses through machine learning data science, then customizes your prep program to you so you get the most effective prep possible.

Try our 5-day full access trial for free:

The last quality a perfect-scoring essay must demonstrate is precision of language and flow in writing . The author of this GRE Analytical Writing sample fulfills this requirement by using language to precisely and economically convey meaning throughout her essay. Here’s one example of precise and effective use of language in the essay:

This disease had ravaged the human race since prehistorical days, and yet with the technology of vaccines, free thinking humans dared to imagine a world free of smallpox.

In this excerpt, the author uses the evocative word “ravaged” to show the dire extent of the problem solved by technology, reinforcing that the issue was previously considered impossible to cope with. She also uses the phrase “humans dared to imagine” in this sentence, which ties the example being discussed back to the previous paragraph’s discussion of human imagination.

While there are a couple of minor errors in this excerpt (“prehistorical” should be “prehistoric,” “free thinking” should be “free-thinking”), they do not significantly change the meaning of the author’s words and so do not detract from the overall effectiveness of the author’s language.

Nope nope nope we've gone too prehistoric, walk it back, folks

Return to Table of Contents

The second of the GRE Issue essay samples I’ll be analyzing is written in response to the following prompt about the values of cooperation vs. competition:

“The best way for a society to prepare its young people for leadership in government, industry, or other fields is by instilling in them a sense of cooperation, not competition.”

Write a response in which you discuss the extent to which you agree or disagree with the claim. In developing and supporting your position, be sure to address the most compelling reasons or examples that could be used to challenge your position.

The sample Issue essay written in response to this topic takes the stance that cooperation, not competition, is a preferable value to instill in young people in preparation for government. You can read the full essay on page 108 of this PDF . Read on for a discussion of the different ways in which this essay meets the requirements for a perfect score.

As with the previous GRE essay sample, we’ll start by looking at how this essay meets the perfect-scoring essay criteria of stating a clear and insightful position (as required by the essay task). The author fulfills the first part of the criteria with his clear statement of his thesis in the last line of the very first paragraph:

I would have to agree that the best way to prepare young people for leadership roles is to instill in them a sense of cooperation.

He reiterates this clear position with the last two sentences of his conclusion:

Getting to be President of the United States or the managing director of a corporation might require you to win some battles, but once you are there you will need diplomacy and people-skills. Those can be difficult to learn, but if you do not have them, you are likely to be a short-lived leader.

To achieve a perfect Issue essay score, however, it’s not just enough to be clear in your position; your position must also demonstrate insight into the issue . The author of this essay accomplishes this second part by choosing a two-pronged approach to answering the essay question. Rather than merely explaining how cooperativeness leads to positive outcomes in government, industry, and other fields, the author also explains how competitiveness leads to negative outcomes.

Thus, the author makes his position clear by stating it in the opening and closing paragraphs of the essay and shows insight by taking the more complex position that not only is cooperation good, but competition is bad.

body_cooperationovercompetition

The next of the rubric criteria we’ll discuss has to do with how well the author develops his position with examples and reasoning . A great example of this development can be found in the second paragraph of this essay, which discusses the drawbacks of competition.

The author begins his discussion of competitiveness by arguing that it’s a quality that doesn’t need to be “instilled” because it’s already present. Beginning with general reasoning about human behaviors at school and the office to introduce his point, the author then neatly segues into specific examples of competitiveness gone amok (Hitler in Germany and the recent economic meltdown in America).

With each example presented in the essay, the author pushes his position along a little further. He moves from discussing the most extreme historical cases (genocide) to more recent events (economic recession), concluding by focusing in on one person’s life and career (Tiger Woods). This final example allows the author to reach his final destination in his discussion of competitiveness: yes, competition can serve people well up to a certain point, but the price is that it is also “detrimental and ultimately quite destructive.”

Competition is particularly destructive if you're playing chess with glass pieces!

The third way this essay meets the requirements of a perfect-scoring essay is through the logical connection of ideas within and between paragraphs . The transition between the end of paragraph two and the beginning of paragraph three provides a stellar example of this skillful connecting of ideas:

It [competitiveness] served him well in some respects, but it also proved to be detrimental and ultimately quite destructive.

Leaders who value cooperation, on the other ahnd, have historically been less prone to these overreaching, destructive tendencies.

On the face of it, the author only connects the two paragraphs by using a transition phrase (“on the other hand”) that sets up the next paragraph as contrasting with what came before. While this kind of transition would be good enough for a lower-scoring essay, though, the author does not just leave the connection between the two paragraphs at that. Instead, he also connects the two paragraphs by keeping the focus on the same issue from the end of one paragraph to the beginning of the next.

The content-level transition between paragraphs occurs when the author transitions from discussing the “detrimental and ultimately quite destructive” competitiveness of Tiger Woods directly into claiming that cooperation-valuing leaders are “less prone to these overreaching, destructive tendencies.” This twofold linkage of content (deeper level) and transition phrase (more surface level) makes it clear to the reader that the discussion of leaders valuing cooperation follows logically the discussion of negative outcomes for competition-valuing leaders.

THERE CAN BE ONLY OOOOOOOOOOOOONE!

The final 6-level quality demonstrated by this GRE Writing sample is its use of skillful and precise language to convey specific meaning . Overall, the language in this essay is formal and academic , despite the profligate use of first person point of view by the author (which can make writing seem less formal). The following sentence exemplifies the author’s command of language:

The recent economic meltdown was caused in no large part by the leaders of American banks and financial institutions who were obsessed with competing for the almighty dollar.

Despite the minor error in this sentence (it should read “in no small part,” rather than “in no large part,”), the author’s meaning is absolutely clear: competition led to the meltdown. Strong vocabulary choices like “economic meltdown,” “obsessed,” “almighty dollar” are what make this an effective statement of the author’s position. Compare the above excerpt to a more milquetoast version of the same statement:

The recent economic downturn was mostly caused by financial leaders who wanted to earn lots of money.

Want to improve your GRE score by 7+ points?

Check out our best-in-class online GRE prep program . We guarantee your money back if you don't improve your GRE score by 7 points or more.

PrepScholar GRE is entirely online, and it customizes your prep program to your strengths and weaknesses . We also feature 2,000 practice questions , official practice tests, 150 hours of interactive lessons, and 1-on-1 scoring and feedback on your AWA essays.

Check out our 5-day free trial now:

This second sentence has the same basic meaning as the real excerpt from the essay. Because it doesn’t use particularly precise or compelling language, however, this watered-down version ends up minimizing the magnitude of problems caused by competitiveness (which undercuts the author’s point). This vaguer version of the essay excerpt also lacks the word “competing,” which makes it useless as an instance of competition among leaders leading to negative consequences.

The original excerpt from the essay, and indeed the entire GRE essay example, is so strong precisely because it manages to pack in specific relevant language that adds to, rather than detracts from, the author’s meaning.

body_relevantlanguage

The next essay I’ll be analyzing is written in response to the following “Analyze an Argument” prompt:

In surveys Mason City residents rank water sports (swimming, boating and fishing) among their favorite recreational activities. The Mason River flowing through the city is rarely used for these pursuits, however, and the city park department devotes little of its budget to maintaining riverside recreational facilities. For years there have been complaints from residents about the quality of the river’s water and the river’s smell. In response, the state has recently announced plans to clean up Mason River. Use of the river for water sports is therefore sure to increase. The city government should for that reason devote more money in this year’s budget to riverside recreational facilities.

Write a response in which you examine the stated and/or unstated assumptions of the argument. Be sure to explain how the argument depends on the assumptions and what the implications are if the assumptions prove unwarranted.

The GRE Argument essay sample I’ll be analyzing critiques the numerous assumptions made and ultimately concludes that the argument for spending more money on Mason City’s riverside recreational facilities rests on faulty assumptions.

The full text of this essay can be found on the ETS website . Be sure to read through the essay first before coming back to read my analysis of it. We’ll start by looking at the ways in which this GRE essay sample identifies and examines the argument given in the prompt in an insightful way:

There are three key assumptions made by the argument that are identified in the essay:

#1 : The survey results are valid and representative

#2 : The reason Mason River isn’t used is because of odor and pollution

#3 : Cleaning the pollution in the river will get rid of the odor and then lead to more usage by residents

The Argument essay example we’re looking at examines each of the assumptions by considering the implications if the assumptions made by the article turn out not to be true . Here’s part of the essay’s investigation of the second assumption listed above:

Though there have been complaints, we do not know if there have been numerous complaints from a wide range of people, or perhaps from one or two individuals who made numerous complaints.

The author identifies the assumption that complaints indicate many people want to use the river and examines it by reasoning through possible scenarios other than the one presented in the prompt. The insight comes from the fact that the specific possibilities discussed by the author are highly plausible alternative explanations for the facts that would change the validity of the prompt’s assumption. It’s very possible that the complaints were not made by every single resident, or even a majority of residents, as the prompt seems to assume, but were in fact only made by a few people.

As a result of her analysis, the author ultimately concludes that there is insufficient information to support the assumption that Mason River isn’t used due to its odor and pollution.

Bear with me.

The next way the author of this sample GRE essay fulfills the requirements of a perfect-scoring Argument essay is by providing comprehensive support for each of her main points . Throughout the essay, the author is able to explain exactly why each assumption made is problematic by using examples that precisely illustrate her argument.

Consider how this is approached in the second paragraph of the essay. The author starts the paragraph by presenting the assumption made in the essay argument that the survey results can be relied upon. She then proceeds to decimate that assumption with multiple examples of ways in which the survey could be flawed and not be an accurate representation of the residents’ opinions, as can be seen in the following excerpt:

For example, the survey could have asked residents if they prefer using the river for water sports or would like to see a hydroelectric dam built, which may have swayed residents toward river sports. The sample may not have been representative of city residents, asking only those residents who live upon the river. The survey may have been 10 pages long, with 2 questions dedicated to river sports. We just do not know.

The thoroughness of the author’s support for her point is magnified by the specificity of the scenarios she proposes . Stating “the survey might not have been representative of the city residents” would have been far less compelling a point than stating “[t]he sample may not have been representative of city residents, asking only those residents who live upon the river.”

Probably not quite so on the river as this, though.

Another important ideal a perfect-scoring Argument essay must live up to is being organized logically, with clear transitions between ideas . The author of this GRE essay sample is able to meet the first part of this requirement with a simple five-paragraph organizational structure : an introduction, one paragraph for each assumption discussed, and a conclusion.

Accomplishing the logical connection and development of ideas throughout the essay requires a little bit more finesse, but the author still manages it. Here’s an example from the beginning of the third paragraph of a skillful transition:

Additionally, the author implies that residents do not use the river for swimming, boating, and fishing, despite their professed interest, because the water is polluted and smelly.

In the above example, the author uses the transition word “additionally” to connect the ideas that will follow with what went before. The example also references the previous paragraph’s discussion of the unreliability of the survey of residents (“their professed interest”) and links it to the current discussion of pollution and smell being the cause of low participation in riverside recreational activities. The combination of these two methods of connecting the two paragraphs results in a smooth logical flow from one idea to the next.

Let your ideas flow. Like the Mason River.

Lastly, a perfect-scoring Argument essay must be precise and effective in its discussion of ideas, with few if any errors . The author of this essay successfully meets this standard by using purposeful language to efficiently and clearly get her point across, as can be seen in this example from paragraph three:

While a polluted, smelly river would likely cut down on river sports, a concrete connection between the resident’s lack of river use and the river’s current state is not effectively made.

The author contrasts the prompt’s assumption (“a polluted, smelly river would likely cut down on river sports”) with the “concrete connection” that is not present. The essay as a whole is not completely devoid of errors (for example, the author writes “afffected” instead of “affected”), but the errors are few and do not have a negative impact on the clarity of the writing.

body_clarityofwriting

The last of the GRE essay examples I’ll be analyzing at is written in response to this “Analyze an Argument” prompt:

The following is taken from a memo from the advertising director of the Super Screen Movie Production Company.

“According to a recent report from our marketing department, during the past year, fewer people attended Super Screen-produced movies than in any other year. And yet the percentage of positive reviews by movie reviewers about specific Super Screen movies actually increased during the past year. Clearly, the contents of these reviews are not reaching enough of our prospective viewers. Thus, the problem lies not with the quality of our movies but with the public’s lack of awareness that movies of good quality are available. Super Screen should therefore allocate a greater share of its budget next year to reaching the public through advertising.”

Write a response in which you discuss what questions would need to be answered in order to decide whether the recommendation and the argument on which it is based are reasonable. Be sure to explain how the answers to these questions would help to evaluate the recommendation.

The essay written in response to this “Analyze an Argument” prompt raises and evaluates questions about how many viewers and reviews of Super Screen productions there actually were, if there is a strong relationship between how movie reviewers and general audiences react to movies, and whether or not the percentage of positive reviews about a movie reflects how much of an impact reviews have on audiences.

The full text of this GRE essay sample can be found on p. 112 of this PDF . Read through the essay first, then check below for an analysis of its positive (and negative) qualities.

The first aspect of the essay we’ll analyze is how it succeeds in identifying and examining the parts of the argument that are relevant to the task . In the essay’s introduction, the author mentions that there are questions that need to be asked (“Before this plan is implemented, however, Super Screen needs to address some questions about its possible flaws”), but he really hammers it home in the conclusion by specifying which questions need to be answered:

In conclusion, there are many questions Super Screen needs to answer before using this advertising director’s plan. They need to look carefully at actual numbers, both of viewership and of positive reviews. The also need to identify the relationship that their target audience has with movie reviewers and determine how their target audience feels about their movies. Fianlly they need to take a nuanced look at the movie reviews that they use in their advertising.

With this conclusion, the author hits the three main points that need to be considered before agreeing to the advertising director’s plan : viewer and review numbers, audience reactions to reviews, and whether or not reviews are a useful metric by which to measure movie success.

An instance of the author identifying a particular argument can be found in the third paragraph of this GRE essay sample. The paragraph starts by clearly stating the question that needs to be answered (what the number of positive reviews was and how it compared to past reviews). After this initial identification of the question, the author also explains how answering this question would have an impact on the usefulness of the recommendation: if the increase in positive reviews was from 1% to 2%, allocating more money to advertising to emphasize this fact is likely to have less impact than if the money were instead budgeted towards improving film quality.

Lights! Camera! And a 200% increase in script quality!

Another quality all perfect-scoring Argument essays must contain is strong and thorough support for each point discussed . The author of the GRE essay sample we’re analyzing fulfills this requirement, supporting every question she raises about the argument in the prompt by showing how its answer would affect the recommendation.

A good example of this all coming together happens in paragraph five of the essay:

Finally the studio must ask whether the percentage of positive reviews is really a relevant way to measure the potential impact of movie reviews. There are dozens of movie reviewers but when deciding whether to not to go to a movie, the general public will usually pick from among the 10 most popular movie reviews. These are the reviews that will impress the public if they are included in advertising. If the most popular movie reviewers disliked Super Screen movies that a larger number of small time film bloggers reviewed positively, Super Screen needs to think of a new advertising strategy.

In this paragraph, the author opens by identifying the element of argument to be discussed (are positive reviews a useful way to measure the impact of movie reviews in general?). She then develops this point through reasoning about why the answer to this question might contradict the assumption made in the argument (people mostly use popular reviews to decide on what movies to see, rather than the ratio of popular to negative reviews).

The author ends this paragraph by conclusively showing that the answer to the question raised in this paragraph is crucial for determining whether or not Super Screen should follow the advertising director’s plan: if the percent of positive reviews isn’t a good way to measure movie impact and the real issue is that relatively few popular movie reviewers liked Super Screen movies, then the recommendation of the advertising department is unreasonable.

No amount of advertising's going to wake up that movie reviewer.

The third requirement for a perfect-scoring Argument essay is that it must develop and connect ideas in a clear and logical fashion. The organization of this GRE argument essay sample helps accomplish this by routing the author’s thoughts into an introduction, four body paragraphs, and a conclusion . Each body paragraph of the essay is centered around one or two related questions. A good example of this can be found in paragraph four, which contains two related questions about the relationship between audiences and movie reviewers:

Finally, Super Screen needs to ask what the relationship is between its viewers and the movie reviewers cited in the memo. Using a survey distributed to its target audience, Super Screen could determine if movie reviews have an effect on their audience’s decision to go see a movie, whether movie reviewers tended to have the same taste as the target audience and exactly whether or not movie reviews are reaching the audience. Super Screen also needs to consider how its movie choices have affected the separate movie reviewer and audience populations. If the studio has switched from making mega- blockbuster action movies to more nuanced dramas, the general public may be less willing to go see their movies even though movie critics prefer the dramas to the action movies.

The above paragraph starts out by discussing if Super Screen’s target audiences are affected by reviews and whether their audiences and movie reviewers have the same taste, then segues into discussing if the studio’s film-making choices have affected audiences and movie reviews. The transition between the two different questions being discussed is effected by the simple use of the word “also” in the third sentence of the paragraph:

Super Screen also needs to consider how its movie choices have affected the separate movie reviewer and audience populations. [bolded for emphasis]

The last sentence of the paragraph again links back to the discussion of audience taste vs. reviewer taste, reinforcing the close and logical connection between the two questions discussed in the paragraph.

Bo Gordy-Stith/Flickr

Finally, a perfect-scoring Argument essay must employ varied and precise language, with few errors . Earlier, we discussed paragraph four as a particularly strong example of the author’s effective development of ideas. The last sentence of this paragraph contributes to this efficacy through the use of specific language :

“If the studio has switched from making mega-blockbuster action movies to more nuanced dramas, the general public may be less willing to go see their movies even though movie critics prefer the dramas to the action movies.”

The use of the descriptor “mega-blockbuster” to describe the action movies preferred by the masses effectively conjures up something that is the diametric opposite of a “nuanced drama.” In addition, the author’s contrasting of the “mega-blockbuster action movies” with “more nuanced dramas” parallels the second half of the sentence’s contrasting of the preferences of the general public vs. those of the (possibly) more refined movie reviewer.

There are a few minor spelling errors (e.g. in “attendence” instead of “attendance”), and the last two body paragraphs both start with “finally” (which is a little repetitive), but in general, this is a skillfully written essay. It’s not perfectly polished like an essay you’d turn in for school, but that’s absolutely OK. In the grand scheme of the GRE essay scoring rubric, writing flourishes matter much less than clarity of thought and precision of language.

Some Super Screen movie reviewers. MCAD Library/Flickr.

6 Tips for a Perfect-Scoring GRE Essay

To wrap up this article, I’ll go over some of the key points you should take from the four GRE sample essays I analyzed in this article.

#1: Include an Introduction and a Conclusion

One thing that all these perfect-scoring GRE sample essays had in common was an introduction and a conclusion . It doesn’t have to be a full paragraph, but you need to at the very least introduce your ideas at the beginning of your essay and wrap up your conclusions at the end of it.

#2: State Your Position Clearly

In my notes to myself on one of the GRE Issue essay examples I analyzed above, I observed that the author “states her thesis early and often” because of the way her position was made clear throughout the essay. While obviously you don’t want to just repeat the same sentence over and over again, it is imperative that you include at least one clear statement of your position in your essay , preferably in your introduction paragraph.

The importance of clearly stating your position varies between the two GRE essay tasks somewhat. For the Argument essay, you might be able to get away with a vague summary of the points you’ll cover and still get a 4.0 or above on the essay; by contrast, it’s nearly impossible to get above a 3.0 on the Issue essay if you do not clearly state your position on the issue, as that is integral to the essay task itself.

Whatever the prompt or essay type, if you want to get a perfect score on your essay, you’ll need to include a clear statement of your position on the issue or what points you’ll be analyzing in regards to the argument in the prompt.

#3: Be Specific in Your Support

All of the perfect-scoring GRE essay examples analyzed in this article contained specific and relevant support for the claims made by the authors. In the Issue essay examples, the authors drew upon well-defined examples and concise examples that directly supported the author’s position on the issue. In the Argument essay samples, the authors focused in on several specific parts of the arguments and debated their validity using specific hypothetical scenarios and questions.

The takeaway of this for your own writing is that the specific is always more persuasive than the general when it comes to supporting a point. And if you can’t find specific support for your position or for the flaw you’ve found in an argument, then that’s a good sign that you need to consider changing your position or finding another part of the argument to critique.

If you can't support your thesis with specific examples, you might need to find a new thesis.

#4: Explain Your Support Clearly

As I discussed in my analyses of the four GRE Writing samples, whether or not your writing is polished and perfectly worded and spelled is not nearly as important as your successful communication of your ideas and how they are supported . In the GRE essay, all is precision, and analyses of issues that use clearly-explained compelling examples or analyses of arguments that cut to the very heart of why an argument is flawed with supporting explanations will ultimately score higher than beautifully crafted but logically imprecise essays.

#5: Use Transitions

All of the authors of the GRE essay examples analyzed in this article are able to maintain focus and organization in their essays by employing multi-level transitions that link ideas between and within paragraphs on both content and linguistic levels. In your own writing, be conscious of when you are changing from discussing one idea to another and make sure the transition is smooth. Even just adding transition words like “additionally” or “in contrast” to the beginning of new ideas can help your writing flow better.

#6: Stay Organized

While all of the GRE essay examples used in this article were written in response to different prompts, they all adhered to basically the standard five-paragraph , introduction-body paragraphs-conclusion format.

There’s no reason to take extra time away from your analysis of the questions to figure out a unique organizational structure for each essay when the five paragraph essay will get it done just as well (if not better). This is not because other forms are not possible; as the ETS website says, “You are free to organize and develop your response in any way you think will enable you to effectively communicate your ideas about the issue.”

But the utility of the five paragraph form is that it’s a tried-and-true way to keep your essay organized . Using it will save you the time of having to figure out a new organizational strategy for every essay you write. And the more consistently you stick to a simple (but clear) organizational structure, the faster you’ll get at it, until organizing your thoughts logically comes as second-nature (especially important in a timed essay environment when every second counts).

body_what'snext

What’s Next?

Now you know what it takes to get a perfect essay score. But do you actually need to get a perfect 6.0 on GRE Writing? Find out with our discussion of what a good GRE Writing score is .

Curious about how the criteria mentioned in this article translate into numerical scores? Read our article on how the GRE essay is scored to learn more!

Need to boost your essay score quickly? We have 15 great tips and strategies that help you improve your Analytical Writing score .

Ready to dive into practice essays with some practice topics? Use our guide to the 328 official GRE essay topics to get started.

Ready to improve your GRE score by 7 points?

gre issue essay pool ets

Author: Laura Staffaroni

Laura graduated magna cum laude from Wellesley College with a BA in Music and Psychology, and earned a Master's degree in Composition from the Longy School of Music of Bard College. She scored 99 percentile scores on the SAT and GRE and loves advising students on how to excel and fulfill their college and grad school dreams. View all posts by Laura Staffaroni

gre issue essay pool ets

Greg Mat

.

gre issue essay pool ets

  • GRE Exam Info
  • What's Tested on the GRE?
  • What's a Good GRE Score?
  • Taking the GRE
  • GRE Study Plans
  • GRE Tips and Tricks
  • Applying to Grad School
  • GRE Question of the Day
  • GRE Pop Quiz
  • GRE 20-Minute Workout
  • Free GRE Practice Test
  • GRE Practice Packs
  • GRE Classes

GRE Issue Essay: Practice Sample Prompts

Gre issue essay sample prompt #1.

1. The emergence of the online “blogosphere” and social media has significantly weakened the quality of political discourse in the United States. Reason: When anyone can publish political opinions easily, standards for covering news and political topics will inevitably decline.

Write a response in which you examine your own position on the statement. Explore the extent to which you either agree or disagree with it, and support your reasoning with evidence and/or examples. Be sure to reflect on ways in which the statement might or might not be true, and how this informs your thinking on the subject.

GRE Issue Essay Sample Prompt #2

2. Some people believe it is imperative for individuals living in developed nations to reduce their energy consumption and lead a more sustainable lifestyle, given the evidence for global climate change. Others believe that such drastic lifestyle changes are unwarranted, based on the existing evidence for global climate change.

Write a response in which you discuss which view more closely aligns with your own position and explain your reasoning for the position you take. In developing and supporting your position, you should address both of the views presented.

GRE Issue Essay Sample Prompt #3

3. So long as they are aware of the dangers involved, adults should not be legally bound to use seat belts.

Write a response in which you discuss the extent to which you agree or disagree with the statement and explain your reasoning for the position you take. In developing and supporting your position, you should consider ways in which the statement might or might not hold true and explain how these considerations shape your position.

You might also like

What's Tested on the GRE: Analytical Writing

Call 1-800-KAP-TEST or email [email protected]

Prep for an Exam

MCAT Test Prep

LSAT Test Prep

GRE Test Prep

GMAT Test Prep

SAT Test Prep

ACT Test Prep

DAT Test Prep

NCLEX Test Prep

USMLE Test Prep

Courses by Location

NCLEX Locations

GRE Locations

SAT Locations

LSAT Locations

MCAT Locations

GMAT Locations

Useful Links

Kaplan Test Prep Contact Us Partner Solutions Work for Kaplan Terms and Conditions Privacy Policy CA Privacy Policy Trademark Directory

Get the Reddit app

This subreddit is for discussion of the GRE (Graduate Record Examination). If you're studying for the GRE, or can help people who are studying for the GRE, you're in the right place!

AWA POOL IN THE ETS WEBSITE

Will awa argument and issues always come from the pool in the ets website..!

ARGUMENT ESSAY POOL

ISSUE ESSAY POOL

By continuing, you agree to our User Agreement and acknowledge that you understand the Privacy Policy .

Enter the 6-digit code from your authenticator app

You’ve set up two-factor authentication for this account.

Enter a 6-digit backup code

Create your username and password.

Reddit is anonymous, so your username is what you’ll go by here. Choose wisely—because once you get a name, you can’t change it.

Reset your password

Enter your email address or username and we’ll send you a link to reset your password

Check your inbox

An email with a link to reset your password was sent to the email address associated with your account

Choose a Reddit account to continue

IMAGES

  1. Issue-pool

    gre issue essay pool ets

  2. Gre Essay Help

    gre issue essay pool ets

  3. Pin on Quick Saves

    gre issue essay pool ets

  4. Pool of GRE Argument Essay Topics (With images)

    gre issue essay pool ets

  5. Writing the gre argument essay step by step guide

    gre issue essay pool ets

  6. How to Use the GRE Essay Pool for Effective Practice

    gre issue essay pool ets

COMMENTS

  1. GRE General Test Analytical Writing Analyze an Issue Task

    Prepare for the GRE Analytical Writing Analyze an Issue Task. Find tips, sample essay responses with explanation, and the pool of Issue topics.

  2. PDF issue-pool

    Pool of Issue Topics This page contains the Issue topics for the Analytical Writing section of the GRE® General Test. When you take the test, you will be presented with one Issue topic from this pool.

  3. PDF Pool of Analytical Writing Topics

    Pool of Analytical Writing Topics This contains the topics for the Analytical Writing section of the GRE® General Test. When you take the test, you will be presented with one topic from this pool. Each topic consists of an issue statement or statements followed by specific task instructions that tell you how to respond to the issue.

  4. 328 Official GRE Essay Topics to Practice With

    Looking for official GRE essay topics to practice with? Find the full Issue and Argument pools here, plus analysis of the prompts and helpful study tips.

  5. PDF Sample Responses and Reader Commentaries for Analytical Writing Prompts

    The Analytical Writing portion of the GRE consists of two writing topics, an Issue topic and an Argument topic. Analyze an Issue Sample Issue Topic Directions Directions: The Analytical Writing portion of the GRE consists of two writing topics: Analyze an Issue and Analyze an Argument. For this section, Analyze an Issue will be the writing topic.

  6. How to Use the GRE Essay Pool for Effective Practice

    To prepare effectively for this section, it's essential to practice with various essay topics. ETS, the organization that administers the GRE, provides an extensive pool of essay topics for both the Issue Essay and the Argument Essay.

  7. PDF Analytical Writing Sample Essays and Commentaries Large Print (18 point

    The Analytical Writing portion of the GRE®General Test consists of two writing topics, an Issue topic and an Argument topic. This document contains the writing topics for Practice Test #3, the scoring guides for each section, and sample responses with commentaries for each topic. Note: Sample responses are reproduced exactly as written ...

  8. GRE Issue Essay: Strategies + 8 Real Student Essays with Scores

    GRE Issue Essay: Strategies + 8 Real Student Essays with Scores. The content in this post applies in 2024 to the new, shorter GRE! When you sit down at the computer on test day, the very first thing you'll encounter is the GRE AWA Issue essay. For a lot of test-takers, this will feel daunting.

  9. 4 Top-Scoring GRE Sample Essays, Analyzed (Issue + Argument)

    Looking for GRE sample essays? Our guide includes in depth analysis of 4 GRE essay examples, plus take-away tips on how craft a high-scoring essay.

  10. PDF Pool of Issue Topics

    Pool of Issue Topics This page contains the Issue topics for the Analytical Writing section of the GRE® revised General Test. When you take the test, you will be presented with one Issue topic from this pool.

  11. GRE General Test Analytical Writing Overview

    Learn about GRE Analytical Writing, practice with published topic pools, find sample essays and more.

  12. Recommended Practice Topics for the GRE Issue Task

    Recommended Practice Topics for the GRE Issue Task When you take the GRE, you'll have to write two essays for the Analytical Writing section. All the topics are online right now at the Official GRE website. You should pick out a few and practice .

  13. GRE Pool Essay Topics

    GRE sample issue pool essay topic 26. The best way to teach - whether as an educator, employer, or parent - is to praise positive actions and ignore negative ones. Write a response in which you discuss the extent to which you agree or disagree with the claim. In developing and supporting your position, be sure to address the most compelling ...

  14. Feedback on AWA Essay

    I have included an essay prompt and my response to it. The prompt was taken from the publicly available topic pool of ETS: https://www.ets.org/pdfs/gre/analytical ...

  15. GRE Issue Essay: Practice Sample Prompts

    GRE Issue Essay: Practice Sample Prompts The GRE Issue Essay will provide a brief quotation on an issue of general interest and instructions on how to respond. You must evaluate the issue and develop an argument with support for your side of the issue. In the GRE Issue Essay, you will be developing your own argument.

  16. This is the GRE issue essay from ETS Issue Pool, Page no 4.

    This is the GRE issue essay from ETS Issue Pool, Page no 4. In any situation, progress requires discussion among people who have contrasting points of view. Write a response in which you discuss the extent to which you agree or disagree with the statement and explain your reasoning for the position you take.

  17. GRE General Test Analytical Writing Analyze an Argument Task

    Prepare for the GRE Analytical Writing Analyze an Argument Task. Find tips, sample essay responses with explanation, and the pool of Issue topics.

  18. Resource of examples/key points of essay prompts from the ETS ...

    So after relatively finishing my preparation for the Verbal & Quant sections, I've begun getting ready for the AWA essay, and I found that the ETS site provides a pdf pool of writing topics, from which one will appear on the actual exam.

  19. PDF argument-pool

    (For GRE administrations before September 22, 2023) This page contains the Argument topics for the Analytical Writing section of the GRE® General Test. When you take the test, you will be presented with one Argument topic from this pool.

  20. Was this never in the issue essay pool of ETS AWA?

    I was taking the princeton review's practice test and found this prompt. I wrote the essay and while i was trying to see what other people might have written on the same topic afterwards, i couldnt find much (except one or two). then, when i went through the ETS's issue essay pool, i couldnt find the prompt. i'm just wondering was this in their prompt before the GRE was revised and now they ...

  21. PDF Sample GRE ® Issue Task with Strategies, Sample Essay Responses and

    Here is a sample Issue task that you might encounter on the GRE Analytical Writing measure: As people rely more and more on technology to solve problems, the ability of humans to think for themselves will surely deteriorate. Discuss the extent to which you agree or disagree with the statement and explain your reasoning for the position you take ...

  22. How many are the issue essay prompts in the ETS pool?

    How many are the issue essay prompts in the ETS pool? Has anyone counted them? Thinking of checking them out before my GRE exam because I want to provide some pertinent examples. 1 comment Best Add a Comment GreenlightTestPrep • Tutor/Expert/Prep company • 2 yr. ago

  23. AWA POOL IN THE ETS WEBSITE : r/GRE

    Yes, your essay prompts will come from those pools. The top of the Issue pool reads: "This page contains the Issue topics for the Analytical Writing section of the GRE® General Test. When you take the test, you will be presented with one Issue topic from this pool." Will awa argument and issues always come from the pool in the ets website..!